Room-temperature superconductivity?

Point out news stories, on the net or in mainstream media, related to polywell fusion.

Moderators: tonybarry, MSimon

johanfprins
Posts: 708
Joined: Tue Jul 06, 2010 6:40 pm
Location: Johannesbutg
Contact:

Post by johanfprins »

tomclarke wrote:Anyone who wants to see the mistake in Johan's incredibly complex Lorentz Transform post above should note only the sign of the Doppler effect.

Johan is only considering the (1+v/c) component of the Doppler (inbound journey) when he has to consider both inbound and outbound. Look through all his equations. See any (1-v/c)?
It seems that you cannot read: I have said that on the return journey the Doppler factor changes from (1+v/c) tp (1-v/c). If you want to, I can derive the full set of equations also for the return journey, but it should be obvious to anybody that can do high school algebra that except for the change in the Doppler factor, you will also on the returm leg have that both twins will receive the exact same number of pulses.
But it is clear from the diagrams (or personal observation of say train whistles)
Sound waves have a uniquely stationary medium within which they propagate. Light waves have not. Thus to compare the two types of Doppler shifts is showing collossal ignorance on your part.

If he did that, he would have to deal with the fact that between the two twins the received inbound part would start at a different time, which is the source of the asymmetry.
The results above, derived directly from the Lorentz transformation, do not agree with your obviously wrong reasoning when sending light pulses.

tomclarke
Posts: 1683
Joined: Sun Oct 05, 2008 4:52 pm
Location: London
Contact:

Post by tomclarke »

johanfprins wrote:
tomclarke wrote: it should be obvious to anybody that can do high school algebra that except for the change in the Doppler factor, you will also on the returm leg have that both twins will receive the exact same number of pulses.
Perhaps you could enlighten us. Looking at the diagrams in my post, and the reasoning afterwards, it would seem that just after the moving twin has started the return leg, the stationary twin will still be receiving low frequency pulses, transmitted from the moving twin on its outward leg, whereas the moving twin will immediately receive high frequency pulses once it has turned round.

This by the way is true for any similar problem, it is not a relativistic effect exclusively.

tomclarke
Posts: 1683
Joined: Sun Oct 05, 2008 4:52 pm
Location: London
Contact:

Post by tomclarke »

Johan wrote: Sound waves have a uniquely stationary medium within which they propagate. Light waves have not. Thus to compare the two types of Doppler shifts is showing collossal ignorance on your part.
The comparison I was making was the way in which high/low frequencies received depend on the relative velocities. The precise details are different, I agree, but you will notice I use the correct relativistic frequencies, and also note that this is different in the non-relativistic case.

tomclarke
Posts: 1683
Joined: Sun Oct 05, 2008 4:52 pm
Location: London
Contact:

Post by tomclarke »

Johan wrote: The results above, derived directly from the Lorentz transformation, do not agree with your obviously wrong reasoning when sending light pulses.
Indeed, because you assume that the "return leg" pulses are the same for both moving and stationary twin, whereas it is obvious that the change due to the moving twin turnaround takes some time to propagate to the stationary twin, but affects moving twin immediately.

happyjack27
Posts: 1439
Joined: Wed Jul 14, 2010 5:27 pm

Post by happyjack27 »

tomclarke wrote:
happyjack27 wrote:
tomclarke wrote:To show the twins must have different ages, all you need is Doppler effect, and Einstein's 2nd postulate - which Johan has been silent about.

....

(8) Doing the calculation for the number of pulses from each twin:

Stationary twin: f1T1 + f2T2
Moving twin: f1'T1'+f2'T2'
Let us also label total time:
T1 + T2 = T1' + T2' = T

(9)
f2 = f2' = (1+v/c)f
f1 = f1' = (1-v/c)f
[f is the frequency at which pulses are emitted. It will turn out we need to scale this by a time dilation factor. But that will be the result of our calculation, we do NOT assume this]

Difference in age is: f(1-v/c)(T1-T1') + f(1+v/c)(T2-T2') =
f[ (T - T) - v/c( T2' - T2) - v/c(T1- T1') ]

The first term cancels and (by inspection) T2' > T2, T1 > T1' so we must have the stationary twin older than the moving twin.
this is all fine and good math, but which of the two twins is the "stationary" one? you are sure to get a different answer from each twin!
The left-hand twin is stationary (the two pictures are the same situation, drawn twice to show light rays in different directions).

I do get a different answer for the stationary and moving twin, as I show.

You can't reverse the argument because it depends on one twin turning round. This turnaround point alters the Doppler shift for the moving twin immediately, for the stationary twin only after light has travelled to it from the moving twin. Obviously, only the moving twin turns around!

Of course, if both twins turnaround (and move at v/2) the problem is symmetric and there is no difference.
the problem is symmetric.

twin A reads his/her current time as t_x and twin B's time as t_y.

likewise,

twin B reads his/her current time as t_x and twin A's time as t_y.

this appears to be a paradox, but the paradox is resolved when one realizes that there is a transform function between between twin B's time and twin A, and vice versa. hence:

t_x = t_a = t_b, and
t_y = F(a,b,t_b) = F(b,a,t_a)

-----------------------

what it sounds like you have done with you "turn around" thing is accidently created a triplets problem, rather than a twins problem. here, you introduce a third observable at an arbitrary point, who, through his own movements and spins, is able to introduce asymmetry.

that, or you are applying extension to points. what you have then is two line segments or vectors, and thus FOUR points, not two. (or if you are only concerned with orientation and use polar coordinates, 6 coordinates, not 4.) and you have an entirely different problem.

tomclarke
Posts: 1683
Joined: Sun Oct 05, 2008 4:52 pm
Location: London
Contact:

Post by tomclarke »

happyjack27 wrote:
tomclarke wrote:
happyjack27 wrote: this is all fine and good math, but which of the two twins is the "stationary" one? you are sure to get a different answer from each twin!
The left-hand twin is stationary (the two pictures are the same situation, drawn twice to show light rays in different directions).

I do get a different answer for the stationary and moving twin, as I show.

You can't reverse the argument because it depends on one twin turning round. This turnaround point alters the Doppler shift for the moving twin immediately, for the stationary twin only after light has travelled to it from the moving twin. Obviously, only the moving twin turns around!

Of course, if both twins turnaround (and move at v/2) the problem is symmetric and there is no difference.
the problem is symmetric.

twin A reads his/her current time as t_x and twin B's time as t_y.

likewise,

twin B reads his/her current time as t_x and twin A's time as t_y.
That is symmetric, but when one twin turns around it affects his counting of other twin's time immediately, and otehr twin is so affected only after some time. the turnaroudn is a real (asymmetric) physical event. It has consequences for Doppler effect.
this appears to be a paradox, but the paradox is resolved when one realizes that there is a transform function between between twin B's time and twin A, and vice versa. hence:

t_x = t_a = t_b, and
t_y = F(a,b,t_b) = F(b,a,t_a)
I don't see how you can make assumptions like that without some extra stuff. I'm not sure whether thsi is correct, would need definitions of all your terms first. My proof rests only on c is constant.
what it sounds like you have done with you "turn around" thing is accidently created a triplets problem, rather than a twins problem. here, you introduce a third observable at an arbitrary point, who, through his own movements and spins, is able to introduce asymmetry.
No, the movemnet of one of the twins chnages, the movement of the other does not.
that, or you are applying extension to points. what you have then is two line segments or vectors, and thus FOUR points, not two.
I'm not sure what is unclear about my diagrams. they illustrate, vividly, how the Doppler frequency change happens assymetrically between the two twins. try having a look?
(or if you are only concerned with orientation and use polar coordinates, 6 coordinates, not 4.) and you have an entirely different problem.
OK, please go back to my original post and say which of my steps you disagree with, then I can help resolve this.

The problem is asymmetric, because one twin changes velocity and the other does not. This changed velocity alters Doppler. But of course it alters Doppler on the moving twin immediately, and on the stationary twin only after the change has propagated.

happyjack27
Posts: 1439
Joined: Wed Jul 14, 2010 5:27 pm

Post by happyjack27 »

tomclarke wrote:
happyajck27 wrote:
t_x = t_a = t_b, and
t_y = F(a,b,t_b) = F(b,a,t_a)
I don't see how you can make assumptions like that without some extra stuff. I'm not sure whether thsi is correct, would need definitions of all your terms first. My proof rests only on c is constant.
that is where you are wrong. your proof rests on quite a few more assumptions than c being constant.

also, i am not making an assumption, i am REMOVING one by introducing F.

i gave the neccessary and sufficient definitions for t_x & t_y above. t_a is a's intrinsic time, t_b is b's. "a" is all available information about a, as "b" is for b. F is undefined. F is the functional that we would try to deduce given empirical data about the other variables. (in SR it turns out to be related to the lorentz transform, but our argument only requires that F(a,b,x) is not necessarily equal to x. which is, as i said, REMOVING the hidden assumption that F(a,b,x) neccessarily equals x, and thus that e.g. a's measurement of b's time is neccesarily the same as b's intrinsic time. )
tomclarke wrote: ...
The problem is asymmetric, because one twin changes velocity and the other does not. This changed velocity alters Doppler. But of course it alters Doppler on the moving twin immediately, and on the stationary twin only after the change has propagated.
see, therein lies your problem: when you say a twin "changes velocity", you don't specify what coordinate system/reference frame it changes velocity with respect to.
Last edited by happyjack27 on Wed Jan 11, 2012 6:04 pm, edited 1 time in total.

johanfprins
Posts: 708
Joined: Tue Jul 06, 2010 6:40 pm
Location: Johannesbutg
Contact:

Post by johanfprins »

tomclarke wrote:
johanfprins wrote:
tomclarke wrote: it should be obvious to anybody that can do high school algebra that except for the change in the Doppler factor, you will also on the returm leg have that both twins will receive the exact same number of pulses.
Perhaps you could enlighten us. Looking at the diagrams in my post,
The diagrams in your post are irrelevant since they contradict the Lorentz transformation.
it would seem that just after the moving twin has started the return leg, the stationary twin will still be receiving low frequency pulses
Firstly there are no "stationary" and "moving" twins: Only two twins moving relative to one another with a speed v. Each twin is under then impression that he is stationary and that the other twin is moving.
transmitted from the moving twin on its outward leg, whereas the moving twin will immediately receive high frequency pulses once it has turned round.
As soon as the twins move towards one another, they still receive the same number of pulses, notwithstanding the fact that these pulses are now following one another at a faster rate. Eqs. [4] and [10] are still valid except that now the factor (1+v/c) is replaced with (1-v/c). But twin1 still receives the light pulse from twin2 at exactly the same time that twin2 receives the light pulse from twin1. AND THE DIRECT DERIVATION USING THE LT CONFIRMS THAT IT MUST BE SO!

johanfprins
Posts: 708
Joined: Tue Jul 06, 2010 6:40 pm
Location: Johannesbutg
Contact:

Post by johanfprins »

tomclarke wrote:
Johan wrote: Sound waves have a uniquely stationary medium within which they propagate. Light waves have not. Thus to compare the two types of Doppler shifts is showing collossal ignorance on your part.
The comparison I was making was the way in which high/low frequencies received depend on the relative velocities. The precise details are different, I agree, but you will notice I use the correct relativistic frequencies, and also note that this is different in the non-relativistic case.
But you reach a conclusion that contradicts what is correctly derived from the Lorentz transformation. So obviously you are WRONG!

johanfprins
Posts: 708
Joined: Tue Jul 06, 2010 6:40 pm
Location: Johannesbutg
Contact:

Post by johanfprins »

tomclarke wrote:
Johan wrote: The results above, derived directly from the Lorentz transformation, do not agree with your obviously wrong reasoning when sending light pulses.
Indeed, because you assume that the "return leg" pulses are the same for both moving and stationary twin, whereas it is obvious that the change due to the moving twin turnaround takes some time to propagate to the stationary twin, but affects moving twin immediately.
I assumed nothing except that the twins are each sending out a light pulse every second on their synchronised clocks. That is all I assume.

I then use the Lorentz transformation to prove that, although the time of pulse arrival at each twin is slower on the outbound trip than on the inbound trip, both twins receive exactly the same number of pulses during the whole trip. Therefore their clocks MUST show the same time when they meet up again.

tomclarke
Posts: 1683
Joined: Sun Oct 05, 2008 4:52 pm
Location: London
Contact:

Post by tomclarke »

johanfprins wrote:
tomclarke wrote: Sorry, what intertial reference frame?
Both twins are each within an inertial reference frame and they each send out light pulses from their reference frames.
I am not involing reference frames here. All I consider is the Doppler effect on light emitted by one twin and received by the other.
So you are saying that the twins are not each within an inertial reference frame and that the two light sources are not each stationary within each of the twin's inertial refrence frames.
No I am just saying i do not need the reference frame concept for this proof.
The hwole theory of SR is based on the concept on inertial refrence frames moving relative to one another. How can you make any concluions by not involving refrence frames?
If you read through my post's steps, which clearly you have never done, you would see how.
why is it asymmetric? Just look at the diagrams! Consider the light-ray sent from the moving twin at turaround.
Both twins are moving relative to the other twin and they both send light pulses to one another at turnaround. There is not a unique stationary twin. This is the whole point of Special Relativity that there is not a unquely stationary reference frame.
Sigh. The "stationary" twin is so called because his movement is inertial and therefore there exists an inertial frame in which he is always stationary. The moving twin has no such frame. It is a shorthand to call them stationary and moving.

You may if you like consider my whole proof to be carries out in the rest frame of the stationary twin.
This is received by the stationary twin AFTER turnaround and marks the start of T1.
The times at which each twin receives the other twin's signal after the other twin has sent the signal is shorter on the return journey owing to the Doppler shift, but for each pulse that twin1 receives twin2 also receives a pulse at the same exact time.
That is clearly not true, look at the diagrams to see why, or my many explanations: the "return journey" effect on received pulses starts at different times for each twin.
Thus the total number of pulses received by both twins is still the same once they meet up.
See above.
The mathematics derived from the Lorentz transformation confirms that this is so.
You apply the LT assuming that the inward and outward segments are the same at both ends, hence your wrong answer. maths of LT is irrelevant to this.
By (time) symmetry, T2' = T1'
therefore T1 < T2 (by the time it takes light to travel between the twins when they are furthest separated).
Look at equation [4] and [10] above. They do not agree with your assertion here at all. Where do you say have I made a mistake when I derived these equations?
Johan. I have stated clearly my arguments. I have also stated what is the (initial and basic) defect in yours. Therefore instead of asking me to say what is wrong with your argument (I already have) try to engage with mine. If you are correct there will be some step in it which you can question.
These equations remain true in all frames, which is why I do not need to use them, and why you need to anwer this question of the inherent asymmetry simply.
As I have proved by using the Lorentz transformation, there is no asymmetry invloved when you do the mathematics correctly.
That is a silly statement, see above.
Intuitively, the asymmetry derives from the fcat that one twin turns around, the other does not.
Their speed is relative: You can only conclude that one turns around as seen from a third reference frame. But this is nonsensical, since one twin sees the other twin to "turn around" and other twin sees the first twin as turning around. The turn around is symmetrical.
Physically the turnaround is asymmetrical. Relativity applies to inertial frame, not to frames which suddenly reverse direction. that is one of your fundamental mistakes.
The turnaround alters the Doppler effect,
Correct, it does: My impeccable mathematics using the Lorentz equations prove that it makes no difference.
See above
Light going to the twin which turns around will have different doppler effect immediately after the turnaround. Light coming from it will have different Doppler effect at time of turnaround + light travel time.
This does NOT change the fact as proved above, that the two twins must receive exactly the same number of pulses during the whole journey.
Ahhh... Most revealing. You admit this is true. You see that it proves the asymmetry. You have however no answer to this except to illogically assert that it makes no difference to your stated (without reasons) result. Of course it does, because it chnages the number of pulses received by the stationary twin at the lower frequency, and the number at the higher frequency.
Let me do the calculation by using the Lorentz transformation:

...snip...

Do you now see how dangerous it is to ignore the Lorentz transformation
Wow. You were the one saying I misused mystical LTs instead of good-old einstein's postulates!
You are distorting what I have said. I said that when you derive a result, for example that the twins will age at diffrent rates, you are violating Einstein's postulates since according to these postulates and the Lorentz transformation derived from it, the twins cannot age at different rates.
I'll correct your incorrect use of LTs here after you have dealt with the (simpler) issue of Doppler asymmetry.

My equations have dealt with it. What eklse must I do.
For example, you could go through the steps of my original post saying which is the first one you disagree with, and why. That is the normal way in which arguments are resolved.
Why wait? "Correct" me it if you can: Which I doubt. You cannot correct which is already correct. The Doppler effect is part of the Lorentz transformation and does appear within my equations as (1+v/c) on the outbound leg and as (1-v/c) on the return leg but the equations still prove compellingly that the two twins will each receive the SAME number of pulses during the whole journey.
I don't see that can be when your equations do not address the issue, obvious from my diagrams, that the length of pulses received at higher frequency is shorter on the stationary twin than the moving twin. In your derivation you have just assumed the situation is symmetrical.

Your stated reasons, a distorted diagram in which the change in velocity of one twin cannot be seen, and an apparent change in velocity of the other twin is added which does not exist, have been explained by me.

This distorted diagram does not contain the information needed to determine whether a given light ray from the stationary to the moving twin is doppler shifted up or down, because you cannot see on it where the moving twin chnages velocity.

My reasons, the two diagrams I give and the points in my post, have not been addressed by you.
Therefore tgeir respective clopcks MUST show the time for the whole journey
I have not used Minkowski anything. My diagrams are simple time/position diagrams such as are commonly used to illustrate moving objects.
Exactly! And such a diagram is nonsense when the Lorentz transformation applies.
No, such diagrams are exactly what is used to prove the Lorentz transform from Einstein postulates, which I thought was what you would want, given that you claim "first principles" theory superior to that others derive.

tomclarke
Posts: 1683
Joined: Sun Oct 05, 2008 4:52 pm
Location: London
Contact:

Post by tomclarke »

johanfprins wrote:
tomclarke wrote:
Johan wrote: The results above, derived directly from the Lorentz transformation, do not agree with your obviously wrong reasoning when sending light pulses.
Indeed, because you assume that the "return leg" pulses are the same for both moving and stationary twin, whereas it is obvious that the change due to the moving twin turnaround takes some time to propagate to the stationary twin, but affects moving twin immediately.
I assumed nothing except that the twins are each sending out a light pulse every second on their synchronised clocks. That is all I assume.

I then use the Lorentz transformation to prove that, although the time of pulse arrival at each twin is slower on the outbound trip than on the inbound trip, both twins receive exactly the same number of pulses during the whole trip. Therefore their clocks MUST show the same time when they meet up again.
Johan,

HOW do you determine when the pulse frequency increases for each twin?

Note that one twin changes direction to mark start of inbound and end of outbound journey.

Note this could happen at any time

Note causality requires that the (Doppler) effect of this change cannot propagate to other twin faster than light.

Note that (Doppler) effect of this change will immediately alter pulses on the twin that changes velocity.

tomclarke
Posts: 1683
Joined: Sun Oct 05, 2008 4:52 pm
Location: London
Contact:

Post by tomclarke »

johanfprins wrote:
tomclarke wrote:
Johan wrote: Sound waves have a uniquely stationary medium within which they propagate. Light waves have not. Thus to compare the two types of Doppler shifts is showing collossal ignorance on your part.
The comparison I was making was the way in which high/low frequencies received depend on the relative velocities. The precise details are different, I agree, but you will notice I use the correct relativistic frequencies, and also note that this is different in the non-relativistic case.
But you reach a conclusion that contradicts what is correctly derived from the Lorentz transformation. So obviously you are WRONG!

Ahhhh....

Even more revealing!

Do you realise what you are saying here. I produce a watertight argument, which you cannot disprove. Instead, you resort to "I know your argument is wrong because my arguments tell me something contradictory".

That is the reponse of a very very poor physicist. It is what would lead some people to reject the results of the Michelson-Morley experiments since they were so difficult to reconcile with existing thought.

You will notice I have never used this type of argument on you, and you have always used it on me. When I disagree with your arguments I give the steps in the argument which I believe are wrong, and why.

happyjack27
Posts: 1439
Joined: Wed Jul 14, 2010 5:27 pm

Post by happyjack27 »

this is a mess. tom, you are not even seeing the arguments made by others or the flaws in yours that others point out because you are so confident that you are right it is blinding you.

happyjack27
Posts: 1439
Joined: Wed Jul 14, 2010 5:27 pm

Post by happyjack27 »

[quote=tomclarke]
You may if you like consider my whole proof to be carries out in the rest frame of the stationary twin.
[/quote]

again, which twin is stationary?!

by even saying this you are missing the entire point of Relativity!

Post Reply